53 LR Two Question 6
- « 53 LR Two Question 5
- 1510 of 3815
- 53 LR Two Question 7 »
Comments

Permalink Submitted by bergaran2 on Apr 24 2016
Why can't it be answer choice A or E? I feel like all three answer choices ruin the argument I just don't get why D is the right choice?

Permalink Submitted by majorgeneraldave on Apr 25 2016
(A) is incorrect because the author did provide justification for that belief—that she hid in a "well-concealed" blind.
(E) is incorrect because even if it were true, it would not hurt the argument—in fact, it would strengthen her claim that the earlier belief is now incorrect!
Clearer?